GATE CSE Engineering Mathematics PYQ

GATE CSE Engineering Mathematics PYQ

Q1➡ | GATE 2021 Set-1
Let p and q be two propositions. Consider the following two formulae in propositional logic.

Which one of the following choices is correct?
i ➥ S1 is a tautology but S2 is not a tautology.
ii ➥ Both S1 and S2 are tautologies.
iii ➥ Neither S1 and S2 are tautology.
iv ➥ S1 is not a tautology but S2 is a tautology.

Show Answer With Best Explanation

Answer: I
Explanation: Upload Soon

More Discussion Explanation On YouTube Propositional LogicHelp-Line

Q2➡ | GATE 2021 Set-1
Consider the following expression

The value of the above expression (rounded to 2 decimal places) is____

Show Answer With Best Explanation

Answer: 0.25
Explanation: Upload Soon

More DiscussionExplanation On YouTubeCalculus Help-Line

Q3➡ | GATE 2021 Set-1
In an undirected connected planar graph G, there are eight vertices and five faces. The number of edges in G is _____.

Show Answer With Best Explanation

Answer: 11
Explanation: Upload Soon

More DiscussionExplanation On YouTubeGraph Theory Help-Line

Q4➡ | GATE 2021 Set-1
Let G = (V, E) be an undirected unweighted connected graph. The diameter of G is defined as:

Let M be the adjacency matrix of G.
Define graph G2 on the same set of vertices with adjacency matrix N, where

Which one of the following statements is true?
i ➥
ii ➥
iii ➥
iv ➥

Show Answer With Best Explanation

Answer: I
Explanation: Upload Soon

More DiscussionExplanation On YouTubeGraph Theory Help-Line

Q5➡ | GATE 2021 Set-1
Consider the two statements.
S1: There exist random variables X and Y such that (E[(X-E(X)) (Y-E(Y))])2>Var[X] Var[Y]
S2: For all random variables X and Y, Cov[X,Y]=E[|X-E[X]| |Y-E[Y]|]
Which one of the following choices is correct?
i ➥ Both S1 and S2 are false.
ii ➥ S1 is false, but S2 is true.
iii ➥ Both S1 and S2 are true.
iv ➥ S1 is true, but S2 is false.

Show Answer With Best Explanation

Answer: I
Explanation: Upload Soon

More DiscussionExplanation On YouTubeProbability and stattisticsHelp-Line

Q6➡ | GATE 2021 Set-1
Let G be a group of order 6, and H be a subgroup of G such that 1 < |H| < 6. Which one of the following options is correct?
i ➥ G may not be cyclic, but H is always cyclic.
ii ➥ Both G and H may not be cyclic.
iii ➥ Both G and H are always cyclic.
iv ➥ G is always cyclic, but H may not be cyclic.

Show Answer With Best Explanation

Answer: I
Explanation: Upload Soon

More DiscussionExplanation On YouTubeGroup Theory Help-Line

Q7➡ | GATE 2021 Set-1
A relation R is said to be circular of aRb and bRc together imply cRa. Which of the following options is/are correct?
i ➥ If a relation S is circular and symmetric, then S is an equivalence relation.
ii ➥ If a relation S is transitive and circular, then S is an equivalence relation.
iii ➥ If a relation S is reflexive and symmetric, then S is an equivalence relation.
iv ➥ If a relation S is reflexive and circular, then S is an equivalence relation.

Show Answer With Best Explanation

Answer: IV
Explanation: Upload Soon

More DiscussionExplanation On YouTubeSet Theory Help-Line

Q8➡ | GATE 2021 Set-1
An articulation point in a connected graph is a vertex such that removing the vertex and its incident edges disconnects the graph into two or more connected components. Let T be a DFS tree obtained by doing DFS in a connected undirected graph G. Which of the following options is/are correct?
i ➥ A leaf of T can be an articulation point in G.
ii ➥ If u is an articulation point in G such that x is an ancestor of u in T and y is a descendent of u in T, then all paths from x to y in G must pass
through u.
iii ➥ Root of T is an articulation point in G if and only if it has 2 or more children
iv ➥ Root of T can never be an articulation point in G.

Show Answer With Best Explanation

Answer: III
Explanation: Upload Soon

More DiscussionExplanation On YouTubeBFS and DFS Help-Line

Q9➡ | GATE 2021 Set-1
A sender (S) transmits a signal, which can be one of the two kinds: H and L with probabilities 0.1 and 0.9 respectively, to a receiver (R). In the graph below, the weight of edge (u, v) is the probability of receiving v when u is transmitted, where u, v ∈ {H, L}. For example, the probability that the received signal is L given the transmitted signal was H, is 0.7.

If the received signal is H, the probability that the transmitted signal was H (rounded to 2 decimal places) is ________.

Show Answer With Best Explanation

Answer: 0.04
Explanation: Upload Soon

More DiscussionExplanation On YouTubeProbability and statistics Help-Line

Q10➡ | GATE 2021 Set-1
Consider the following matrix.

The largest eigenvalue of the above matrix is___________

Show Answer With Best Explanation

Answer: 3
Explanation: Upload Soon

More DiscussionExplanation On YouTubeLinear Algebra Help-Line

Q11➡ | GATE 2021 Set-2
Let G be a connected undirected weighted graph.
Consider the following two statements.
S1: There exists a minimum weighted edge in G which is present in every minimum spanning tree of G.
S2:If every edge in G has distinct weight, then G has a unique minimum spanning tree.
Which of the following options is correct?
i ➥ S1 is false and S2 is true.
ii ➥ Both S1 and S2 are true.
iii ➥ Both S1 and S2 are false.
iv ➥ S1 is true and S2 is false.

Show Answer With Best Explanation

Answer: I
Explanation: Upload Soon

More DiscussionExplanation On YouTubeMinimim Spanning TreeHelp-Line

Q12➡ | GATE 2021 Set-2
Choose the correct choice(s) regarding the following propositional logic assertion S:
i ➥ S is a tautology.
ii ➥ The antecedent of S is logically equivalent to the consequent of S.
iii ➥ S is a contradiction.
iv ➥ S is neither a tautology nor a contradiction.

Show Answer With Best Explanation

Answer: I, II
Explanation: Upload Soon

More DiscussionExplanation On YouTubePropositional LogicHelp-Line

Q13➡ | GATE 2021 Set-2
i ➥ There does not exist an injunction from S1 to S2.
ii ➥ There exists a bijection from S1 to S2.
iii ➥ There does not exist a bijection from S1 to S2.
iv ➥ There exists a surjection from S1 to S2.

Show Answer With Best Explanation

Answer: II, IV
Explanation: Upload Soon

More DiscussionExplanation On YouTubeFunction Help-Line

Q14➡ | GATE 2021 Set-2

Show Answer With Best Explanation

Answer: 19
Explanation: Upload Soon

More DiscussionExplanation On YouTubeTopic Help-Line

Q15➡ | GATE 2021 Set-2
Suppose that P is a 45 matrix such that every solution of the equation Px=0 is a scalar multiple of
[2 5 4 3 1]T. The rank of P is _____.

Show Answer With Best Explanation

Answer: 4
Explanation: Upload Soon

More DiscussionExplanation On YouTubeC Programming Help-Line

Q16➡ | GATE 2021 Set-2
For a given biased coin, the probability that the outcome of a toss is a head is 0.4. This coin is tossed 1,000 times. Let X denote the random variable whose value is the number of times that head appeared in these 1,000 tosses. The standard deviation of X (rounded to 2 decimal places) is ______.

Show Answer With Best Explanation

Answer: 15.00 to 16.00
Explanation: Upload Soon

More DiscussionExplanation On YouTubeProbability Help-Line

Q17➡ | GATE 2021 Set-2
In an examination, a student can choose the order in which two questions (QuesA and QuesB) must be attempted.
• If the first question is answered wrong, the student gets zero marks.
• If the first question is answered correctly and the second question is not answered correctly, the student gets the marks only for the first question.
• If both the questions are answered correctly, the student gets the sum of the marks of the two questions.
The following table shows the probability of correctly answering a question and the marks of the question respectively.

Assuming that the student always wants to maximize her expected marks in the examination, in which order should she attempt the questions and what is the expected marks for that order (assume that the questions are independent)?
A First QuesB and then QuesA. Expected marks 22.
i ➥ First QuesB and then QuesA. Expected marks 22.
ii ➥ First QuesA and then QuesB. Expected marks 14.
iii ➥ First QuesB and then QuesA. Expected marks 14.
iv ➥ First QuesA and then QuesB. Expected marks 16.

Show Answer With Best Explanation

Answer: IV
Explanation: Upload Soon

More DiscussionExplanation On YouTube Probability Help-Line

Q18➡ | GATE 2021 Set-2
A bag has r red balls and b black balls. All balls are identical except for their colours. In a trial, a ball is randomly drawn from the bag, its colour is noted and the ball is placed back into the bag along with another ball of the same colour. Note that the number of balls in the bag will increase by one, after the trial. A sequence of four such trials is conducted. Which one of the following choices gives the probability of drawing a red ball in the fourth trial?
i ➥
ii ➥
iii ➥
iv ➥

Show Answer With Best Explanation

Answer: III
Explanation: Upload Soon

More DiscussionExplanation On YouTubeProbability Help-Line

Q19➡ | GATE 2021 Set-2
For two n-dimensional real vectors P and Q, the operation s(P, Q) is defined as follows:
i ➥ 9
ii ➥ 10
iii ➥ 100
iv ➥ 11

Show Answer With Best Explanation

Answer: II
Explanation: Upload Soon

More DiscussionExplanation On YouTubeSet Theory Help-Line

Q20➡ | GATE 2021 Set-2
Consider the following directed graph:

Which of the following is/are correct about the graph?
i ➥ The graph does not have a topological order.
ii ➥ The graph does not have a strongly connected component.
iii ➥ A depth-first traversal staring at vertex S classifies three directed edges as back edges.
iv ➥ For each pair of vertices u and v, there is a directed path from u to v.

Show Answer With Best Explanation

Answer: I, III
Explanation: Upload Soon

More DiscussionExplanation On YouTubeGraph Help-Line

Q21➡ | GATE 2021 Set-2
In a directed acyclic graph with a source vertex s, the quality-score of s directed path is defined to be the product of the weights of the edges on the path. Further, for a vertex v other than s, the quality-score of v is defined to be the maximum among the quality-scores of all the paths from s to v. The quality-score of s is assumed to be 1.

The sum of the quality-scores of all the vertices in the graph shown above is _______.

Show Answer With Best Explanation

Answer: 929
Explanation: Upload Soon

More DiscussionExplanation On YouTubeGraph Theory Help-Line

Q22➡ | GATE 2021 Set-2
Let S be a set consisting of 10 elements. The number of tuples of the form (A, B) such that A and B are subsets of S, and A ⊆ B is _______.

Show Answer With Best Explanation

Answer: 59049
Explanation: Upload Soon

More DiscussionExplanation On YouTubeSet Theory Help-Line

Q23➡ | GATE 2020
Consider the functions
I. e-x
II. x2-sin x
III. √(x3+1)
Which of the above functions is/are increasing everywhere in [0,1]?
i ➥ I and III only
ii ➥ II and III only
iii ➥ III only
iv ➥ II only

Show Answer With Best Explanation

Answer: III
Explanation: Upload Soon

More DiscussionExplanation On YouTubeCalculus Help-Line

Q24➡ | GATE 2020
Let G be a group of 35 elements. Then the largest possible size of a subgroup of G other than G itself is ________.

Show Answer With Best Explanation

Answer: 7
Explanation: Upload Soon

More DiscussionExplanation On YouTubeSet Theory Help-Line

Q25➡ | GATE 2020
Let R be the set of all binary relations on the set {1,2,3}. Suppose a relation is chosen from R at random. The probability that the chosen relation is reflexive (round off to 3 decimal places) is ______.

Show Answer With Best Explanation

Answer: 0.125
Explanation: Upload Soon

More DiscussionExplanation On YouTubeSet Theory Help-Line

Q26➡ | GATE 2020
Let A and B be two n×n matrices over real numbers. Let rank(M) and det(M) denote the rank and determinant of a matrix M, respectively. Consider the following statements,
I. rank(AB) = rank(A) rank(B)
II. det(AB) = det(A) det(B)
III. rank(A + B) ≤ rank(A) + rank(B)
IV. det(A + B) ≤ det(A) + det(B)
Which of the above statements are TRUE?
i ➥ I and IV only
ii ➥ III and IV only
iii ➥ II and III only
iv ➥ I and II only

Show Answer With Best Explanation

Answer: III
Explanation: Upload Soon

More DiscussionExplanation On YouTubeLinear Algebra Help-Line

Show Answer With Best Explanation

Answer: 12
Explanation: Upload Soon

Q27➡ | GATE 2020
The number of permutations of the characters in LILAC so that no character appears in its original position, if the two L’s are indistinguishable, is ____.
More DiscussionExplanation On YouTubeCombinatorics Help-Line

Q28➡ | GATE 2020
Which one of the following predicate formulae is NOT logically valid?
Note that W is a predicate formula without any free occurrence of x.
i ➥ ∀x(p(x) ∨ W) ≡ ∀x p(x) ∨ W
ii ➥ ∃x(p(x) ∧ W) ≡ ∃x p(x) ∧ W
iii ➥ ∃x(p(x) → W) ≡ ∀x p(x) → W
iv ➥ ∀x(p(x) → W) ≡ ∀x p(x) → W

Show Answer With Best Explanation

Answer: IV
Explanation: Upload Soon

More DiscussionExplanation On YouTubePropositional Logic Help-Line

Q29➡ | GATE 2020
For n>2, let a ∈ {0,1}n be a non-zero vector. Suppose that x is chosen uniformly at random from {0,1}n. Then, the probability that is an odd number is ________.

Show Answer With Best Explanation

Answer: 0.5
Explanation: Upload Soon

More DiscussionExplanation On YouTubeProbability Help-Line

Q30➡ | GATE 2020
Graph G is obtained by adding vertex s to K3,4 and making s adjacent to every vertex of K3,4. The minimum number of colours required to edge-colour G is ________.

Show Answer With Best Explanation

Answer: 7
Explanation: Upload Soon

More DiscussionExplanation On YouTubeGraph Theory Help-Line

Q31➡ | GATE 2019
Let U = {1,2,…,n}. Let A = {(x,X)|x ∈ X, X ⊆ U}. Consider the following two statements on |A|.

I.

II.
Which of the above statements is/are TRUE?
i ➥ Both I and II
ii ➥ Neither I nor II
iii ➥ Only I
iv ➥ Only II

Show Answer With Best Explanation

Answer: I
Explanation: Upload Soon

More DiscussionExplanation On YouTube Set Theory Help-Line

Q32➡ | GATE 2019
Let G be an undirected complete graph on n vertices, where n > 2. Then, the number of different Hamiltonian cycles in G is equal to
i ➥ (n-1)!
ii ➥ (n-1)! /2
iii ➥ n!
iv ➥ 1

Show Answer With Best Explanation

Answer: II or IV
Explanation: Upload Soon

More DiscussionExplanation On YouTubeGraph Theory Help-Line

Q33➡ | GATE 2019
Let G be an arbitrary group. Consider the following relations on G:
R1: ∀a,b ∈ G, aR1b if and only if ∃g ∈ G such that a = g-1bg
R2: ∀a,b ∈ G, aR2b if and only if a = b-1
Which of the above is/are equivalence relation/relations?
i ➥ R1 only
ii ➥ Neither R1 and R2
iii ➥ R2 only
iv ➥ R1 and R2

Show Answer With Best Explanation

Answer: I
Explanation: Upload Soon

More DiscussionExplanation On YouTube Set Theory Help-Line

Q34➡ | GATE 2019
Let X be a square matrix. Consider the following two statements on X.
I. X is invertible.
II. Determinant of X is non-zero.
Which one of the following is TRUE?
i ➥ II implies I; I does not imply II.
ii ➥ I does not imply II; II does not imply I.
iii ➥ I implies II; II does not imply I.
iv ➥ I and II are equivalent statements.

Show Answer With Best Explanation

Answer: IV
Explanation: Upload Soon

More Discussion Explanation On YouTube Linear Algebra Help-Line

Q35➡ | GATE 2019
Consider Z = X – Y, where X, Y and Z are all in sign-magnitude form. X and Y are each represented in n bits. To avoid overflow, the representation of Z would require a minimum of:
i ➥ n – 1 bits
ii ➥ n + 1 bits
iii ➥ n bits
iv ➥ n + 2 bits

Show Answer With Best Explanation

Answer: II
Explanation: Upload Soon

More DiscussionExplanation On YouTubeNumber Systems Help-Line

Q36➡ | GATE 2019
i ➥ 108/7
ii ➥ Limit does not exist
iii ➥ 1
iv ➥ 53/12

Show Answer With Best Explanation

Answer: I
Explanation: Upload Soon

More DiscussionExplanation On YouTubeCalculus Help-Line

Q37➡ | GATE 2019
Consider the first order predicate formula φ:
∀x[(∀z z|x ⇒ ((z = x) ∨ (z = 1))) ⇒ ∃w (w > x) ∧ (∀z z|w ⇒ ((w = z) ∨ (z = 1)))]
Here ‘a|b’ denotes that ‘a divides b’, where a and b are integers. Consider the following sets:
S1. {1, 2, 3, …, 100}
S2. Set of all positive integers
S3. Set of all integers
Which of the above sets satisfy φ?
i ➥ S2 and S3
ii ➥ S1 and S3
iii ➥ S1 and S2
iv ➥ S1, S2 and S3

Show Answer With Best Explanation

Answer: I
Explanation: Upload Soon

More DiscussionExplanation On YouTubePropositional LogicHelp-Line

Q38➡ | GATE 2019
Let G be any connected, weighted, undirected graph.
I. G has a unique minimum spanning tree, if no two edges of G have the same weight.
II. G has a unique minimum spanning tree, if, for every cut of G, there is a unique minimum-weight edge crossing the cut.

Which of the above statements is/are TRUE?
i ➥ Both I and II
ii ➥ I only
iii ➥ Neither I nor II
iv ➥ II only

Show Answer With Best Explanation

Answer: I
Explanation: Upload Soon

More DiscussionExplanation On YouTubeGraph Theory Help-Line

Q39➡ | GATE 2019
Suppose Y is distributed uniformly in the open interval (1,6). The probability that the polynomial 3x2 + 6xY + 3Y + 6 has only real roots is (rounded off to 1 decimal place) ________.

Show Answer With Best Explanation

Answer: 0.8
Explanation: Upload Soon

More DiscussionExplanation On YouTubeProbability Help-Line

Q40➡ | GATE 2019
Consider the following matrix:

The absolute value of the product of Eigen values of R is ________.

Show Answer With Best Explanation

Answer: 12
Explanation: Upload Soon

More DiscussionExplanation On YouTubeLinear Algebra Help-Line

Q41➡ | GATE 2019
Consider the augmented grammar given below:
S’ → S
S → 〈L〉 | id
L → L,S | S
Let I0 = CLOSURE ({[S’ → ·S]}). The number of items in the set GOTO (I0 , 〈 ) is: ___________.

Show Answer With Best Explanation

Answer: 5
Explanation: Upload Soon

More DiscussionExplanation On YouTubeProbability Help-Line

Q42➡ | GATE 2018
Two people, P and Q, decide to independently roll two identical dice, each with 6 faces, numbered 1 to 6. The person with the lower number wins. In case of a tie, they roll the dice repeatedly until there is no tie. Define a trial as a throw of the dice by P and Q. Assume that all 6 numbers on each dice are equi-probable and that all trials are independent. The probability (rounded to 3 decimal places) that one of them wins on the third trial is_____.

Show Answer With Best Explanation

Answer: 0.21 to 0.24
Explanation: Upload Soon

More DiscussionExplanation On YouTube probability Help-Line

Q43➡ | GATE 2018

Show Answer With Best Explanation

Answer: 0.27 To 0.30
Explanation: Upload Soon

More DiscussionExplanation On YouTubeEngineering Mathematics Help-Line

Q44➡ | GATE 2018

Show Answer With Best Explanation

Answer: 3
Explanation: Upload Soon

More DiscussionExplanation On YouTubeLinear Algebra Help-Line

Q45➡ | GATE 2018
The chromatic number of the following graph is _.

Show Answer With Best Explanation

Answer: 3
Explanation: Upload Soon

More DiscussionExplanation On YouTubeGraph-Theory Help-Line

Q46➡ | GATE 2018
Let G be a finite group on 84 elements. The size of a largest possible proper subgroup of G is_________.

Show Answer With Best Explanation

Answer: 42 To 42
Explanation: Upload Soon

More DiscussionExplanation On YouTubeSet Theory Help-Line

Q47➡ | GATE 2018
Assume that multiplying a matrix G1 of dimension p×q with another matrix G2 of dimension q×r requires pqr scalar multiplications. Computing the product of n matrices G1G2G3…Gn can be done by parenthesizing in different ways. Define GiGi+1 as an explicitly computed pair for a given parenthesization if they are directly multiplied. For example, in the matrix multiplication chain G1G2G3G4G5G6 using parenthesization(G1(G2G3))(G4(G5G6)), G2G3 and G5G6 are the only explicitly computed pairs.

Consider a matrix multiplication chain F1F2F3F4F5, where matrices F1, F2, F3, F4 and F5 are of dimensions 2×25, 25×3, 3×16, 16×1 and 1×1000, respectively. In the parenthesization of F1F2F3F4F5 that minimizes the total number of scalar multiplications, the explicitly computed pairs is/ are
i ➥ F1F2 and F3F4 only
ii ➥ F2F3 only
iii ➥ F3F4 only
iv ➥ F1F2 and F4F5 only

Show Answer With Best Explanation

Answer: III
Explanation: Upload Soon

More DiscussionExplanation On YouTubeLinear Algebra Help-Line

Q48➡ | GATE 2018
Let G be a simple un-directed graph. Let TD be a depth first search tree of G. Let TB be a breadth first search tree of G. Consider the following statements.
(I) No edge of G is a cross edge with respect to TD (A cross edge in G is between two nodes neither of which is an ancestor of the other in TD
(II) For every edge (u,v) of G, if u is at depth i and v is at depth j in TB , then ∣i−j∣ = 1.
Which of the statements above must necessarily be true?
i ➥ I only
ii ➥ II only
iii ➥ Both I and II
iv ➥ Neither I nor II

Show Answer With Best Explanation

Answer: I
Explanation: Upload Soon

More DiscussionExplanation On YouTubeGraphs Help-Line

Q49➡ | GATE 2018
Consider the first-order logic sentence
φ ≡ ∃s∃t∃u∀v∀w∀x∀y ψ(s,t,u,v,w,x,y)
where ψ(s,t,u,v,w,x,y) is a quantifier-free first-order logic formula using only predicate symbols, and possibly equality, but no function symbols. Suppose φ has a model with a universe containing 7 elements.
Which one of the following statements is necessarily true?
i ➥ There exists at least one model of φ with universe of size less than or equal to 3.
ii ➥ There exists no model of φ with universe of size less than or equal to 3
iii ➥ There exists no model of φ with universe of size greater than 7.
iv ➥ Every model of φ has a universe of size equal to 7.

Show Answer With Best Explanation

Answer: I
Explanation: Upload Soon

More DiscussionExplanation On YouTube Propositional Logic Help-Line

Q50➡ | GATE 2018
Let N be the set of natural numbers. Consider the following sets,

P: Set of Rational numbers (positive and negative)
Q: Set of functions from {0, 1} to N
R: Set of functions from N to {0, 1}
S: Set of finite subsets of N

Which of the above sets are countable?
i ➥ Q and S only
ii ➥ P and S only
iii ➥ P and R only
iv ➥ P, Q and S only

Show Answer With Best Explanation

Answer: IV
Explanation: Upload Soon

More DiscussionExplanation On YouTubeLinear Algebra Help-Line

Q51➡ | GATE 2018
Consider a matrix P whose only eigenvectors are the multiples of
Consider the following statements.
(I) P does not have an inverse
(II) P has a repeated eigenvalue
(III) P cannot be diagonalized
Which one of the following options is correct?
i ➥ Only I and II are necessarily true
ii ➥ Only II and III are necessarily true
iii ➥ Only I and III are necessarily true
iv ➥ Only II is necessarily true

Show Answer With Best Explanation

Answer: II
Explanation: Upload Soon

More DiscussionExplanation On YouTube Linear Algebra Help-Line

Q52➡ | GATE 2018
Let G be a graph with 100! vertices, with each vertex labeled by a distinct permutation of the numbers 1, 2, …, 100. There is an edge between vertices u and v if and only if the label of u can be obtained by swapping two adjacent numbers in the label of v. Let y denote the degree of a vertex in G, and z denote the number of connected components in G.
Then, y + 10z =______.

Show Answer With Best Explanation

Answer: 109
Explanation: Upload Soon

More DiscussionExplanation On YouTubeGraph TheoryHelp-Line

Q53➡ | GATE 2018
Consider Guwahati (G) and Delhi (D) whose temperatures can be classified as high (H), medium (M) and low (L). Let P(HG) denote the probability that Guwahati has high temperature. Similarly, P(MG) and P(LG) denotes the probability of Guwahati having medium and low temperatures respectively. Similarly, we use P(HD), P(MD) and P(LD) for Delhi.
The following table gives the conditional probabilities for Delhi’s temperature given Guwahati’s temperature.

Consider the first row in the table above. The first entry denotes that if Guwahati has high temperature (HG) then the probability of Delhi also having a high temperature (HD) is 0.40; i.e., P(HD ∣ HG) = 0.40. Similarly, the next two entries are P(MD ∣ HG) = 0.48 and P(LD ∣ HG) = 0.12. Similarly for the other rows.
If it is known that P(HG) = 0.2, P(MG) = 0.5, and P(LG) = 0.3, then the probability (correct to two decimal places) that Guwahati has high temperature given that Delhi has high temperature is_____.

Show Answer With Best Explanation

Answer: 0.60 To 0.62
Explanation: Upload Soon

More DiscussionExplanation On YouTubeLinear Algebra Help-Line

Q54➡ | GATE 2017 Set-1
Let c1, cn be scalars not all zero. Such that where ai are column vectors in Rn. Consider the set of linear equations.
Ax = B.
where A = [a1…….an] and Set of equations has
i ➥ no solution
ii ➥ a unique solution at x = Jn where Jn denotes a n-dimensional vector of all 1
iii ➥ finitely many solutions
iv ➥ infinitely many solutions

Show Answer With Best Explanation

Answer: IV
Explanation: Upload Soon

More DiscussionExplanation On YouTubeLinear Algebra Help-Line

Q55➡ | GATE 2017 Set-1
Consider the first-order logic sentence F: ∀x(∃yR(x,y)). Assuming non-empty logical domains, which of the sentences below are implied by F?
I. ∃y(∃xR(x,y))
II. ∃y(∀xR(x,y))
III. ∀y(∃xR(x,y))
IV. ¬∃x(∀y¬R(x,y))
i ➥ IV only
ii ➥ II and III only
iii ➥ I and IV only
iv ➥ II only

Show Answer With Best Explanation

Answer: III
Explanation: Upload Soon

More DiscussionExplanation On YouTubePropositional Logic Help-Line

Q56➡ | GATE 2017 Set-1
The statement (¬p) ⇒ (¬q) is logically equivalent to which of the statements below?
I. p ⇒ q
II. q ⇒ p
III. (¬q) ∨ p
IV. (¬p) ∨ q
i ➥ I and IV only
ii ➥ I only
iii ➥ II and III only
iv ➥ II only

Show Answer With Best Explanation

Answer: III
Explanation: Upload Soon

More DiscussionExplanation On YouTube Propositional Logic Help-Line

Q57➡ | GATE 2017 Set-1
Let X be a Gaussian random variable with mean 0 and variance σ2. Let Y = max(X, 0) where max(a,b) is the maximum of a and b. The median of Y is ___________.

Show Answer With Best Explanation

Answer: 0.0
Explanation: Upload Soon

More DiscussionExplanation On YouTubeProbability Help-Line

Q58➡ | GATE 2017 Set-1
Let A be m×n real valued square symmetric matrix of rank 2 with expression given below.

Consider the following statements
(i) One eigenvalue must be in [-5, 5].
(ii) The eigenvalue with the largest magnitude must be strictly greater than 5.
Which of the above statements about engenvalues of A is/are necessarily CORRECT?
i ➥ (I) only
ii ➥ Neither (I) nor (II)
iii ➥ (II) only
iv ➥ Both (I) and (II)

Show Answer With Best Explanation

Answer: I
Explanation: Upload Soon

More DiscussionExplanation On YouTubeLinear Algebra Help-Line

Q59➡ | GATE 2017 Set-1
Let u and v be two vectors in R2 whose Euclidean norms satisfy ||u||=2||v||. What is the value of α such that w = u + αv bisects the angle between u and v?
i ➥ -1/2
ii ➥ 2
iii ➥ 1/2
iv ➥ 1

Show Answer With Best Explanation

Answer: II
Explanation: Upload Soon

More DiscussionExplanation On YouTube Linear Algebra Help-Line

Q60➡ | GATE 2017 Set-1
Let p, q and r be prepositions and the expression (p → q) → r be a contradiction. Then, the expression (r → p) → q is
i ➥ a contradiction
ii ➥ always TRUE when q is TRUE
iii ➥ a tautology
iv ➥ always TRUE when p is FALSE

Show Answer With Best Explanation

Answer: II
Explanation: Upload Soon

More DiscussionExplanation On YouTubePropositional Logic Help-Line

Q61➡ | GATE 2017 Set-1
The value of
i ➥ is 1
ii ➥ does not exist
iii ➥ is 0
iv ➥ is -1

Show Answer With Best Explanation

Answer: I
Explanation: Upload Soon

More DiscussionExplanation On YouTubeCalculus Help-Line

Q62➡ | GATE 2017 Set-1
The number of integers between 1 and 500 (both inclusive) that are divisible by 3 or 5 or 7 is __________.

Show Answer With Best Explanation

Answer: 271
Explanation: Upload Soon

More DiscussionExplanation On YouTubeCombinatorics Help-Line

Q63➡ | GATE 2017 Set-2
Let p, q, r denote the statements “It is raining”, “It is cold”, and “It is pleasant”, respectively. Then the statement “It is not raining and it is pleasant, and it is not pleasant only if it is raining and it is cold” is represented by
i ➥ (¬p ∧ r) ∨ (r → (p ∧ q))
ii ➥ (¬p ∧ r) ∧ ((p ∧ q) → ¬r)
iii ➥ (¬p ∧ r) ∨ ((p ∧ q) → ¬r)
iv ➥ (¬p ∧ r) ∧ (¬r → (p ∧ q))

Show Answer With Best Explanation

Answer: IV
Explanation: Upload Soon

More DiscussionExplanation On YouTube Propositional Logic Help-Line

Q64➡ | GATE 2017 Set-2
If f(x) = R sin(πx/2) + S, f'(1/2) = √2 and , then the constants R and S are, respectively.
i ➥
ii ➥
iii ➥
iv ➥

Show Answer With Best Explanation

Answer: III
Explanation: Upload Soon

More DiscussionExplanation On YouTube Calculus Help-Line

Q65➡ | GATE 2017 Set-2
G is an undirected graph with n vertices and 25 edges such that each vertex of G has degree at least 3. Then the maximum possible value of n is __________.

Show Answer With Best Explanation

Answer: 16
Explanation: Upload Soon

More DiscussionExplanation On YouTubeGraph Theory Help-Line

Q66➡ | GATE 2017 Set-2
Let P= and Q= be two matrices.
Then the rank of P+Q is __________.

Show Answer With Best Explanation

Answer: 2
Explanation: Upload Soon

More DiscussionExplanation On YouTube Matrices Help-Line

Q67➡ | GATE 2017 Set-2
Consider the set X = {a,b,c,d e} under the partial ordering
R = {(a,a),(a,b),(a,c),(a,d),(a,e),(b,b),(b,c),(b,e),(c,c),(c,e),(d,d),(d,e),(e,e)}.
The Hasse diagram of the partial order (X,R) is shown below:

The minimum number of ordered pairs that need to be added to R to make (X,R) a lattice is ___________.

Show Answer With Best Explanation

Answer: 0
Explanation: Upload Soon

More DiscussionExplanation On YouTubeSet Theory Help-Line

Q68➡ | GATE 2017 Set-2
For any discrete random variable X, with probability mass function P(X=j)=pj, pj≥0, j∈{0, …, N} and , define the polynomial function

. For a certain discrete random variable Y, there exists a scalar β∈[0,1] such that gy(Z)=(1-β+βz)N. The expectation of Y is
i ➥ Nβ(1 – β)
ii ➥
iii ➥ N(1 – β)
iv ➥ Not expressible in terms of N and β alone

Show Answer With Best Explanation

Answer: II
Explanation: Upload Soon

More DiscussionExplanation On YouTubeProbability Help-Line

Q69➡ | GATE 2017 Set-2
P and Q are considering to apply for job. The probability that p applies for job is 1/4. The probability that P applies for job given that Q applies for the job 1/2 and The probability that Q applies for job given that P applies for the job 1/3.The probability that P does not apply for job given that Q does not apply for the job
i ➥ 4/5
ii ➥ 5/6
iii ➥ 7/8
iv ➥ 11/12

Show Answer With Best Explanation

Answer: I
Explanation: Upload Soon

More DiscussionExplanation On YouTubeProbabilityHelp-Line

Q70➡ | GATE 2017 Set-2
If a random variable X has a Poisson distribution with mean 5, then the expectation E[(X + 2)2] equals ________.

Show Answer With Best Explanation

Answer: 54
Explanation: Upload Soon

More DiscussionExplanation On YouTubeProbability Help-Line

Q71➡ | GATE 2017 Set-2
If the ordinary generating function of a sequence is , then a3 – a0 is equal to __________.

Show Answer With Best Explanation

Answer: 15
Explanation: Upload Soon

More DiscussionExplanation On YouTubeCombinatorics Help-Line

Q72➡ | GATE 2017 Set-2
If the characteristic polynomial of a 3 × 3 matrix M over ℝ (the set of real numbers) is λ3 – 4λ2 + aλ + 30, a ∈ ℝ, and one eigenvalue of M is 2, then the largest among the absolute values of the eigenvalues of M is __.

Show Answer With Best Explanation

Answer: 5
Explanation: Upload Soon

More DiscussionExplanation On YouTubeLinear Algebra Help-Line

Q73➡ | GATE 2016 Set-1
Let an be the number of n-bit strings that do NOT contain two consecutive 1s. Which one of the following is the recurrence relation for an?
i ➥ an = 2an-1 + 2an-2
ii ➥ an = 2an-1 + an-2
iii ➥ an = an-1 + an-2
iv ➥ an = an-1 + 2an-2

Show Answer With Best Explanation

Answer: III
Explanation: Upload Soon

More DiscussionExplanation On YouTubeRecurrence Relation Help-Line

Q74➡ | GATE 2016 Set-1

Show Answer With Best Explanation

Answer: 1
Explanation: Upload Soon

More DiscussionExplanation On YouTube Calculus Help-Line

Q75➡ | GATE 2016 Set-1
A probability density function on the interval [a,1] is given by 1/x2 and outside this interval the value of the function is zero. The value of a is _________.

Show Answer With Best Explanation

Answer: 0.5
Explanation: Upload Soon

More DiscussionExplanation On YouTube Probability Help-Line

Q76➡ | GATE 2016 Set-1
Two eigenvalues of a 3 × 3 real matrix P are (2 + √-1) and 3. The determinant of P is __________.

Show Answer With Best Explanation

Answer: 15
Explanation: Upload Soon

More DiscussionExplanation On YouTube Linear Algebra Help-Line

Q77➡ | GATE 2016 Set-1
The coefficient of x12 in (x3 + x4 + x5 + x6 + …)3 is ____________.

Show Answer With Best Explanation

Answer: 10
Explanation: Upload Soon

More DiscussionExplanation On YouTubeCombinatorics Help-Line

Q78➡ | GATE 2016 Set-1
Consider the recurrence relation a1 = 8, an = 6n2 + 2n + an-1. Let a99 = K × 104. The value of K is _.

Show Answer With Best Explanation

Answer: 198
Explanation: Upload Soon

More DiscussionExplanation On YouTube Combinatorics Help-Line

Q79➡ | GATE 2016 Set-1
A function f:N+ → N+, defined on the set of positive integers N+, satisfies the following properties:
f(n) = f(n/2) if n is even
f(n) = f(n+5) if n is odd
Let R = {i|∃j : f(j)=i} be the set of distinct values that f takes. The maximum possible size of R is ________.

Show Answer With Best Explanation

Answer: 2
Explanation: Upload Soon

More DiscussionExplanation On YouTubeSet Theory Help-Line

Q80➡ | GATE 2016 Set-1
Consider the following experiment.
Step1: Flip a fair coin twice.
Step2: If the outcomes are (TAILS, HEADS) then output Y and stop.
Step3: If the outcomes are either (HEADS, HEADS) or (HEADS, TAILS), then output N and stop.
Step4: If the outcomes are (TAILS, TAILS), then go to Step 1.
The probability that the output of the experiment is Y is (up to two decimal places) __________.

Show Answer With Best Explanation

Answer: 0.33 to 0.34
Explanation: Upload Soon

More DiscussionExplanation On YouTubeProbability Help-Line

Q81➡ | GATE 2016 Set-2
Consider the following expressions:
(i) false
(ii) Q
(iii) true
(iv) P ∨ Q
(v) ¬Q ∨ P
The number of expressions given above that are logically implied by P ∧ (P ⇒ Q) is _________.

Show Answer With Best Explanation

Answer: 4
Explanation: Upload Soon

More DiscussionExplanation On YouTube Propositional Logic Help-Line

Q82➡ | GATE 2016 Set-2
Let f(x) be a polynomial and g(x) = f'(x) be its derivative. If the degree of (f(x) + f(-x)) is 10, then the degree of (g(x) – g(-x)) is _______.

Show Answer With Best Explanation

Answer: 9
Explanation: Upload Soon

More DiscussionExplanation On YouTubeCalculus Help-Line

Q83➡ | GATE 2016 Set-2
The minimum number of colours that is sufficient to vertex-colour any planar graph is _________.

Show Answer With Best Explanation

Answer: 4
Explanation: Upload Soon

More DiscussionExplanation On YouTube Graph Theory Help-Line

Q84➡ | GATE 2016 Set-2
Consider the systems, each consisting of m linear equations in n variables.
I. If m < n, then all such systems have a solution
II. If m > n, then none of these systems has a solution
III. If m = n, then there exists a system which has a solution
Which one of the following is CORRECT?
i ➥ I, II and III are true
ii ➥ Only II and III are true
iii ➥ Only III is true
iv ➥ None of them is true

Show Answer With Best Explanation

Answer: III
Explanation: Upload Soon

More DiscussionExplanation On YouTube Linear Algebra Help-Line

Q85➡ | GATE 2016 Set-2
Suppose that a shop has an equal number of LED bulbs of two different types. The probability of an LED bulb lasting more than 100 hours given
that it is of Type 1 is 0.7, and given that it is of Type 2 is 0.4. The probability that an LED bulb chosen uniformly at random lasts more than 100
hours is __________.

Show Answer With Best Explanation

Answer: 0.55
Explanation: Upload Soon

More DiscussionExplanation On YouTube Probability Help-Line

Q86➡ | GATE 2016 Set-2
Suppose that the eigenvalues of matrix A are 1, 2, 4. The determinant of (A-1)T is _______.

Show Answer With Best Explanation

Answer: 0.125
Explanation: Upload Soon

More DiscussionExplanation On YouTube Boolean Algebra Help-Line

Q87➡ | GATE 2016 Set-2
A binary relation R on ℕ × ℕ is defined as follows: (a,b)R(c,d) if a≤c or b≤d. Consider the following propositions:
• P: R is reflexive
• Q: R is transitive
Which one of the following statements is TRUE?
i ➥ Both P and Q are true.
ii ➥ P is true and Q is false.
iii ➥ P is false and Q is true.
iv ➥ Both P and Q are false.

Show Answer With Best Explanation

Answer: II
Explanation: Upload Soon

More DiscussionExplanation On YouTubeSet Theory Help-Line

Q88➡ | GATE 2016 Set-2
Which one of the following well-formed formulae in predicate calculus is NOT valid?
i ➥ (∀x p(x) ⇒ ∀x q(x)) ⇒ (∃x ¬p(x) ∨ ∀x q(x))
ii ➥ (∃x p(x) ∨ ∃x q(x)) ⇒ ∃x (p(x) ∨ q(x))
iii ➥ ∃x (p(x) ∧ q(x)) ⇒ (∃x p(x) ∧ ∃x q(x))
iv ➥ ∀x (p(x) ∨ q(x)) ⇒ (∀x p(x) ∨ ∀x q(x))

Show Answer With Best Explanation

Answer: IV
Explanation: Upload Soon

More DiscussionExplanation On YouTubePropositional Logic Help-Line

Q89➡ | GATE 2016 Set-2
Consider a set U of 23 different compounds in a Chemistry lab. There is a subset S of U of 9 compounds, each of which reacts with exactly 3 compounds of U. Consider the following statements:
• I. Each compound in U\S reacts with an odd number of compounds.
• II. At least one compound in U\S reacts with an odd number of compounds.
• III. Each compound in U\S reacts with an even number of compounds.
Which one of the above statements is ALWAYS TRUE?
i ➥ Only I
ii ➥ Only II
iii ➥ Only III
iv ➥ None

Show Answer With Best Explanation

Answer: II
Explanation: Upload Soon

More DiscussionExplanation On YouTubeTCP Help-Line

Q90➡ | GATE 2015 Set-1
If g(x) = 1 – x and h(x)= x/x-1 , then g(h(x)) / h(g(x)) is:
i ➥ h(x)/g(x)
ii ➥ -1/x
iii ➥ g(x)/h(x)
iv ➥ x/(1-x)2

Show Answer With Best Explanation

Answer: I
Explanation: Upload Soon

More DiscussionExplanation On YouTube Calculus Help-Line

Q91➡ | GATE 2015 Set-1
.
i ➥
ii ➥ 0
iii ➥ 1
iv ➥ not defined

Show Answer With Best Explanation

Answer: III
Explanation: Upload Soon

More DiscussionExplanation On YouTubeLimit Help-Line

Q92➡ | GATE 2015 Set-1
Which one of the following is NOT equivalent to p↔q?
i ➥ ( ⌉p∨q)∧(p∨ ⌉q)
ii ➥ ( ⌉p∨q)∧(p→q)
iii ➥ ( ⌉p ∧ q) ∨ (p ∧ ⌉q)
iv ➥ ( ⌉p ∧ ⌉q) ∨ (p ∧ q)

Show Answer With Best Explanation

Answer: III
Explanation: Upload Soon

More Discussion Explanation On YouTube Propositional Logic Help-Line

Q93➡ | GATE 2015 Set-1
i ➥ I and III only
ii ➥ II and III only
iii ➥ I,II and III only
iv ➥ I,II and IV only

Show Answer With Best Explanation

Answer: III
Explanation: Upload Soon

More Discussion Explanation On YouTube Set Theory Help-Line

Q94➡ | GATE 2015 Set-1
In the LU decomposition of the matrix ,if the diagonal elements of U are both 1, then the lower diagonal entry l22 of L is

Show Answer With Best Explanation

Answer: 5
Explanation: Upload Soon

More DiscussionExplanation On YouTube Linear Algebra Help-Line

Q95➡ | GATE 2015 Set-1

Show Answer With Best Explanation

Answer: 0.99
Explanation: Upload Soon

More DiscussionExplanation On YouTube Calculus Help-Line

Q96➡ | GATE 2015 Set-1
Suppose L = {p, q, r, s, t} is a lattice represented by the following Hasse diagram:

For any x, y ∈ L, not necessarily distinct, x ∨ y and x ∧ y are join and meet of x, y respectively. Let L3 = {(x,y,z): x, y, z ∈ L} be the set of all ordered triplets of the elements of L. Let pr be the probability that an element (x,y,z) ∈ L3 chosen equiprobably satisfies x ∨ (y ∧ z) = (x ∨ y) ∧ (x ∨ z).
Then
i ➥ pr = 0
ii ➥ pr = 1
iii ➥ 0 < pr ≤ 1/5
iv ➥ 1/5 < pr < 1

Show Answer With Best Explanation

Answer: IV
Explanation: Upload Soon

More DiscussionExplanation On YouTube Set Theory Help-Line

Q97➡ | GATE 2015 Set-1
Consider the operations
f(X, Y, Z) = X’YZ + XY’ + Y’Z’ and g(X′, Y, Z) = X′YZ + X′YZ′ + XY
Which one of the following is correct?
i ➥ Both {f} and {g} are functionally complete
ii ➥ Only {f} is functionally complete
iii ➥ Only {g} is functionally complete
iv ➥ Neither {f} nor {g} is functionally complete

Show Answer With Best Explanation

Answer: II
Explanation: Upload Soon

More DiscussionExplanation On YouTube Set Theory Help-Line

Q98➡ | GATE 2015 Set-1
Let G be a connected planar graph with 10 vertices. If the number of edges on each face is three, then the number of edges in G is __________.

Show Answer With Best Explanation

Answer: 24
Explanation: Upload Soon

More DiscussionExplanation On YouTubeGraph Theory Help-Line

Q99➡ | GATE 2015 Set-1
Let an represent the number of bit strings of length n containing two consecutive 1s. What is the recurrence relation for an?
i ➥ an-2 + an-1 + 2n-2
ii ➥ an-2 + 2an-1 + 2n-2
iii ➥ 2an-2 + an-1 + 2n-2
iv ➥ 2an-2 + 2an-1 + 2n-2

Show Answer With Best Explanation

Answer: I
Explanation: Upload Soon

More DiscussionExplanation On YouTube Reccurence Relation Help-Line

Q100➡ | GATE 2015 Set-1
Let G = (V, E) be a simple undirected graph, and s be a particular vertex in it called the source. For x ∈ V, let d(x)denote the shortest distance in G from s to x. A breadth first search (BFS) is performed starting at s. Let T be the resultant BFS tree. If (u, v) is an edge of G that is not in T, then which one of the following CANNOT be the value of d(u) – d(v) ?
i ➥ -1
ii ➥ 0
iii ➥ 1
iv ➥ 2

Show Answer With Best Explanation

Answer: IV
Explanation: Upload Soon

More DiscussionExplanation On YouTubeBFS Help-Line

Q101➡ | GATE 2015 Set-1

Show Answer With Best Explanation

Answer: -1
Explanation: Upload Soon

More DiscussionExplanation On YouTubeLinear Algebra Help-Line

Q102➡ | GATE 2015 Set-1
Consider the following 2 × 2 matrix A where two elements are unknown and are marked by a and b. The eigenvalues of this matrix are –1 and 7. What are the values of a and b?
i ➥ a=6, b=4
ii ➥ a=4, b=6
iii ➥ a=3, b=5
iv ➥ a=5, b=3

Show Answer With Best Explanation

Answer: IV
Explanation: Upload Soon

More DiscussionExplanation On YouTubeLinear Algebra Help-Line

Q103➡ | GATE 2015 Set-2
Consider the following statements:
S1: If a candidate is known to be corrupt, then he will not be elected.
S2: If a candidate is kind, he will be elected.

Which one of the following statements follows from S1 and S2 as per sound inference rules of logic?
i ➥ If a person is known to corrupt, he is kind
ii ➥ If a person is not known to be corrupt, he is not kind
iii ➥ If a person is kind, he is not known to be corrupt
iv ➥ If a person is not kind, he is not known to be corrupt

Show Answer With Best Explanation

Answer: III
Explanation: Upload Soon

More DiscussionExplanation On YouTube Propositioal Logic Help-Line

Q104➡ | GATE 2015 Set-2
The cardinality of the power set of {0, 1, 2, … 10} is __________.

Show Answer With Best Explanation

Answer: 2048
Explanation: Upload Soon

More DiscussionExplanation On YouTubeSet Theory Help-Line

Q105➡ | GATE 2015 Set-2
Let R be the relation on the set of positive integers such that aRb if and only if a and b are distinct and have a common divisor other than 1. Which one of the following statements about R is true?
i ➥ R is symmetric and reflexive but not transitive
ii ➥ R is reflexive but not symmetric and not transitive
iii ➥ R is transitive but not reflexive and not symmetric
iv ➥ R is symmetric but not reflexive and not transitive

Show Answer With Best Explanation

Answer: IV
Explanation: Upload Soon

More DiscussionExplanation On YouTube Set Theory Help-Line

Q106➡ | GATE 2015 Set-2
The number of divisors of 2100 is ______.

Show Answer With Best Explanation

Answer: 36
Explanation: Upload Soon

More DiscussionExplanation On YouTube Combinatorics Help-Line

Q107➡ | GATE 2015 Set-2
The larger of the two eigenvalues of the matrix is ______.

Show Answer With Best Explanation

Answer: 6
Explanation: Upload Soon

More DiscussionExplanation On YouTube Linear Algebra Help-Line

Q108➡ | GATE 2015 Set-2
The secant method is used to find the root of an equation f(x) = 0. It is started from two distinct estimates xa and xb for the root. It is an iterative procedure involving linear interpolation to a root. The iteration stops if f(xb) is very small and then xb is the solution. The procedure is given below. Observe that there is an expression which is missing and is marked by? Which is the suitable expression that is to be put in place of? So that it follows all steps of the secant method?
Secant
i ➥ xb – (fb–f(xa)) fb /(xb–xa)
ii ➥ xa – (fa–f(xa)) fa /(xb–xa)
iii ➥ xb – (xb–xa) fb /(fb–f(xa))
iv ➥ xa – (xb–xa) fa /(fb–f(xa))

Show Answer With Best Explanation

Answer: III
Explanation: Upload Soon

More DiscussionExplanation On YouTubeSecant mathod Help-Line

Q109➡ | GATE 2015 Set-2
Let f(x) = x -(1/3) and A denote the area of the region bounded bu f(x) and the X-axis, when x varies from -1 to 1.
Which of the following statements is/are TRUE?
I) f is continuous in [-1,1]
II) f is not bounded in [-1,1]
III) A is nonzero and finite
i ➥ II only
ii ➥ III only
iii ➥ II and III only
iv ➥ I, II and III

Show Answer With Best Explanation

Answer: III
Explanation: Upload Soon

More DiscussionExplanation On YouTubeCalculus Help-Line

Q110➡ | GATE 2015 Set-2
Perform the following operations on the matrix
(i) add the third row to the second row
(ii) Subtract the third column from the first column
The determinant of the resultant matrix is ____________.

Show Answer With Best Explanation

Answer: 0
Explanation: Upload Soon

More DiscussionExplanation On YouTubeLinear Algebra Help-Line

Q111➡ | GATE 2015 Set-2
The number of onto function (surjective function) from set X = {1, 2, 3, 4} to set Y = {a, b, c} is _______.

Show Answer With Best Explanation

Answer: 36
Explanation: Upload Soon

More DiscussionExplanation On YouTubeFunction Help-Line

Q112➡ | GATE 2015 Set-2
Let X and Y denote the sets containing 2 and 20 distinct objects respectively and F denote the set of all possible functions defined from X to Y. Let f be randomly chosen from F. The probability of f being one-to-one is ________.

Show Answer With Best Explanation

Answer: 0.95
Explanation: Upload Soon

More DiscussionExplanation On YouTube Function Help-Line

Q113➡ | GATE 2015 Set-2
A graph is self-complementary if it is isomorphic to its complement for all self-complementary graphs on n vertices, n is
i ➥ A multiple of 4
ii ➥ Even
iii ➥ Odd
iv ➥ Congruent to 0 mod 4, or, 1 mod 4

Show Answer With Best Explanation

Answer: IV
Explanation: Upload Soon

More DiscussionExplanation On YouTubeGraph Theory Help-Line

Q114➡ | GATE 2015 Set-2
In a connected graph, a bridge is an edge whose removal disconnects a graph. Which one of the following statements is true?
i ➥ A tree has no bridges
ii ➥ A bridge cannot be part of a simple cycle
iii ➥ Every edge of a clique with size 3 is a bridge (A clique is any complete sub graph of a graph)
iv ➥ A graph with bridges cannot have a cycle

Show Answer With Best Explanation

Answer: II
Explanation: Upload Soon

More DiscussionExplanation On YouTube Graph Theory Help-Line

Q115➡ | GATE 2015 Set-2
Which one of the following well formed formulae is a tautology?
i ➥ ∀x ∃y R(x,y) ↔ ∃y ∀x R(x,y)
ii ➥ (∀x [∃y R(x,y) → S(x,y)]) → ∀x∃y S(x,y)
iii ➥ [∀x ∃y (P(x,y) → R(x,y)] ↔ [∀x ∃y (¬ P(x,y)∨R(x,y)]
iv ➥ ∀x ∀y P(x,y) → ∀x ∀y P(y,x)

Show Answer With Best Explanation

Answer: III
Explanation: Upload Soon

More DiscussionExplanation On YouTubePropositional Logic Help-Line

Q116➡ | GATE 2015 Set-3
Suppose U is the power set of the set S = {1, 2, 3, 4, 5, 6}. For any T ∈ U, let |T| denote the number of element in T and T’ denote the complement of T. For any T, R ∈ U, let TR be the set of all elements in T which are not in R. Which one of the following is true?
i ➥ ∀X ∈ U (|X| = |X’|)
ii ➥ ∃X ∈ U ∃Y ∈ U (|X| = 5, |Y| = 5 and X ∩ Y = ∅)
iii ➥ ∀X ∈ U ∀Y ∈ U (|X| = 2, |Y| = 3 and X \ Y = ∅)
iv ➥ ∀X ∈ U ∀Y ∈ U (X \ Y = Y’ \ X’)

Show Answer With Best Explanation

Answer: IV
Explanation: Upload Soon

More DiscussionExplanation On YouTubeSet Theory Help-Line

Q117➡ | GATE 2015 Set-3
In the given matrix , one of the eigenvalues is 1. The eigenvectors corresponding to the eigenvalue 1 are
i ➥
ii ➥
iii ➥
iv ➥

Show Answer With Best Explanation

Answer: II
Explanation: Upload Soon

More DiscussionExplanation On YouTubeLinear Algebra Help-Line

Q118➡ | GATE 2015 Set-3
The value of is
i ➥ 0
ii ➥ 1/2
iii ➥ 1
iv ➥

Show Answer With Best Explanation

Answer: III
Explanation: Upload Soon

More DiscussionExplanation On YouTubeCalculus Help-Line

Q119➡ | GATE 2015 Set-3
The number of 4 digit numbers having their digits in non-decreasing order (from left to right) constructed by using the digits belonging to the set {1, 2, 3} is _________.

Show Answer With Best Explanation

Answer: 15
Explanation: Upload Soon

More DiscussionExplanation On YouTubeCombinatorics Help-Line

Q120➡ | GATE 2015 Set-3
In a room there are only two types of people, namely Type 1 and Type 2. Type 1 people always tell the truth and Type 2 people always lie. You give a fair coin to a person in that room, without knowing which type he is from and tell him to toss it and hide the result from you till you ask for it. Upon asking, the person replies the following:
“The result of the toss is head if and only if I am telling the truth.”
Which of the following options is correct?
i ➥ The result is head
ii ➥ The result is tail
iii ➥ If the person is of Type 2, then the result is tail
iv ➥ If the person is of Type 1, then the result is tail

Show Answer With Best Explanation

Answer: I
Explanation: Upload Soon

More DiscussionExplanation On YouTubePropositional Logic Help-Line

Q121➡ | GATE 2015 Set-3
The velocity v (in kilometer/minute) of a motorbike which starts from rest, is given at fixed intervals of time t(in minutes) as follows:

The approximate distance (in kilometers) rounded to two places of decimals covered in 20 minutes using Simpson’s 1/3rd rule is ______.

Show Answer With Best Explanation

Answer: 308 to 310
Explanation: Upload Soon

More DiscussionExplanation On YouTubeSimpson’s 1/3rd Rule Help-Line

Q122➡ | GATE 2015 Set-3
If the following system has non-trivial solution,
px + qy + rz = 0
qx + ry + pz = 0
rx + py + qz = 0
then which one of the following options is True?
i ➥ p – q + r = 0 or p = q = –r
ii ➥ p + q – r = 0 or p = –q = r
iii ➥ p + q + r = 0 or p = q = r
iv ➥ p – q + r = 0 or p = –q = –r

Show Answer With Best Explanation

Answer: III
Explanation: Upload Soon

More DiscussionExplanation On YouTubeLinear Algebra Help-Line

Q123➡ | GATE 2015 Set-3
If for non-zero x,
A:

B:
C:
D:
i ➥ A
ii ➥ B
iii ➥ C
iv ➥ D

Show Answer With Best Explanation

Answer: I
Explanation: Upload Soon

More DiscussionExplanation On YouTubeCalculus Help-Line

Q124➡ | GATE 2015 Set-3
Let R be a relation on the set of ordered pairs of positive integers such that ((p,q),(r,s)) ∈ R if and only if p – s = q – r. Which one of the following is true about R?
i ➥ Both reflexive and symmetric
ii ➥ Reflexive but not symmetric
iii ➥ Not reflexive but symmetric
iv ➥ Neither reflexive nor symmetric

Show Answer With Best Explanation

Answer: III
Explanation: Upload Soon

More DiscussionExplanation On YouTubeRelation Help-Line

Q125➡ | GATE 2015 Set-3
Suppose Xi for i = 1, 2, 3 are independent and identically distributed random variables whose probability mass functions are Pr[Xi = 0] = Pr[Xi = 1]=1/2 for i = 1, 2, 3. Define another random variable Y = X1X2 ⊕ X3, where ⊕ denotes XOR.
Then Pr[Y = 0|X3 = 0] =_______,

Show Answer With Best Explanation

Answer: 0.75
Explanation: Upload Soon

More DiscussionExplanation On YouTube Probability Help-Line

Q126➡ | GATE 2014 Set-1
Consider the statement
“Not all that glitters is gold”
Predicate glitters(x) is true if x glitters and predicate gold(x) is true if x is gold. Which one of the following logical formulae represents the above statement?
i ➥ ∀x: glitters(x) ⇒ ¬gold(x)
ii ➥ ∀x: gold(x) ⇒ glitters()
iii ➥ ∃x: gold(x) ∧ ¬glitters(x)
iv ➥ ∃x: glitters(x) ∧ ¬gold(x)

Show Answer With Best Explanation

Answer: IV
Explanation: Upload Soon

More DiscussionExplanation On YouTube Probability Help-Line

Q127➡ | GATE 2014 Set-1
Suppose you break a stick of unit length at a point chosen uniformly at random. Then the expected length of the shorter stick is____________.

Show Answer With Best Explanation

Answer: 0.24 To 0.27
Explanation: Upload Soon

More DiscussionExplanation On YouTubeSet Theory Help-Line

Q128➡ | GATE 2014 Set-1
Let G = (V,E) be a directed graph where V is the set of vertices and E the set of edges. Then which one of the following graphs has the same strongly connected components as G?
i ➥ G1=(V,E1) where E1={(u,v)|(u,v)∉E}
ii ➥ G2=(V,E2 )where E2={(u,v)│(u,v)∈E}
iii ➥ G3=(V,E3) where E3={(u,v)|there is a path of length≤2 from u to v in E}
iv ➥ G4=(V4,E) where V4 is the set of vertices in G which are not isolated

Show Answer With Best Explanation

Answer: II
Explanation: Upload Soon

More DiscussionExplanation On YouTube Graph Theory Help-Line

Q129➡ | GATE 2014 Set-1
Consider the following system of equations:
3x + 2y = 1
4x + 7z = 1
x + y + z = 3
x – 2y + 7z = 0

The number of solutions for this system is _.

Show Answer With Best Explanation

Answer: 1
Explanation: Upload Soon

More DiscussionExplanation On YouTube Linear Algebra Help-Line

Q130➡ | GATE 2014 Set-1
The value of the dot product of the eigenvectors corresponding to any pair of different eigenvalues of a 4-by-4 symmetric positive definite matrix is ________.

Show Answer With Best Explanation

Answer: 0
Explanation: Upload Soon

More DiscussionExplanation On YouTube Linear Algebra Help-Line

Q131➡ | GATE 2014 Set-1
Let the function

where and f'(θ) denote the derivative of f with respect to θ. Which of the following is/are TRUE?
(I) There exists θ ∈ such that f'(θ)=0.
(II) There exists θ ∈ such that f'(θ)≠0.
i ➥ I only
ii ➥ II only
iii ➥ Both I and II
iv ➥ Neither I nor II

Show Answer With Best Explanation

Answer: III
Explanation: Upload Soon

More DiscussionExplanation On YouTubeLinear Algebra Help-Line

Q132➡ | GATE 2014 Set-1
The function f(x) = x sin x satisfies the following equation: fuu(x) + f(x) +t cosx = 0. The
value of t is .

Show Answer With Best Explanation

Answer: -2
Explanation: Upload Soon

More DiscussionExplanation On YouTubeCalculus Help-Line

Q133➡ | GATE 2014 Set-1
A function f(x) is continuous in the interval [0,2]. It is known that f(0) = f(2) = −1 and
f(1) = 1. Which one of the following statements must be true?
i ➥ There exists a y in the interval (0,1) such that f(y) = f(y + 1)
ii ➥ For every y in the interval (0,1), f(y) = f(2− y)
iii ➥ The maximum value of the function in the interval (0,2) is 1
iv ➥ There exists a y in the interval (0,1) such that f(y) = −f(2− y)

Show Answer With Best Explanation

Answer: I
Explanation: Upload Soon

More DiscussionExplanation On YouTubeLinear Algebra Help-Line

Q134➡ | GATE 2014 Set-1
Four fair six-sided dice are rolled. The probability that the sum of the results being 22 is X/1296.
The value of X is .

Show Answer With Best Explanation

Answer: 10
Explanation: Upload Soon

More DiscussionExplanation On YouTube ProbabilityHelp-Line

Q135➡ | GATE 2014 Set-1
A pennant is a sequence of numbers, each number being 1 or 2. An n-pennant is a sequence of numbers with sum equal to n. For example, (1,1,2) is a 4-pennant. The set of all possible 1-pennants is {(1)}, the set of all possible 2-pennants is {(2), (1,1)}and the set of all 3-pennants is {(2,1), (1,1,1), (1,2)}. Note that the pennant (1,2) is not the same as the pennant (2,1). The number of 10- pennants is .

Show Answer With Best Explanation

Answer: 88.9 To 89.1
Explanation: Upload Soon

More DiscussionExplanation On YouTube CombinatoricsHelp-Line

Q136➡ | GATE 2014 Set-1
Let S denote the set of all functions f: {0,1}4 → {0,1}. Denote by N the number of functions from S to the set {0,1}. The value of log2 log2 N is .

Show Answer With Best Explanation

Answer: 16
Explanation: Upload Soon

More DiscussionExplanation On YouTube functionsHelp-Line

Q137➡ | GATE 2014 Set-1
Consider an un-directed graph G where self-loops are not allowed. The vertex set of G is {(i, j):1 ≤ i ≤ 12, 1 ≤ j ≤ 12}. There is an edge between (a, b) and (c, d) if |a − c| ≤ 1 and |b − d| ≤ 1. The number of edges in this graph is .

Show Answer With Best Explanation

Answer: 506
Explanation: Upload Soon

More DiscussionExplanation On YouTubeGraph Theory Help-Line

Q138➡ | GATE 2014 Set-1
An ordered n-tuple (d1, d2,…, dn) with d1 ≥ d2 ≥ ⋯ ≥ dn is called graphic if there exists a simple undirected graph with n vertices having degrees d1, d2,… , dn respectively. Which of the following 6-tuples is NOT graphic?
i ➥ (1, 1, 1, 1, 1, 1)
ii ➥ (2, 2, 2, 2, 2, 2)
iii ➥ (3, 3, 3, 1, 0, 0)
iv ➥ (3, 2, 1, 1, 1, 0)

Show Answer With Best Explanation

Answer: III
Explanation: Upload Soon

More DiscussionExplanation On YouTube Graph Theory Help-Line

Q139➡ | GATE 2014 Set-1
Which one of the following propositional logic formulas is TRUE when exactly two of p, q, and r are TRUE?
i ➥ ((p ↔ q) ∧ r)∨ (p∧ q ∧ ∼ r)
ii ➥ (∼ (p↔ q) ∧ r) ∨ (p ∧ q ∧ ∼ r)
iii ➥ ((p → q) ∧ r)∨ (p ∧ q ∧ ∼ r)
iv ➥ (∼ (p ↔ q) ∧ r) ∧ (p∧ q ∧ ∼ r)

Show Answer With Best Explanation

Answer: II
Explanation: Upload Soon

More DiscussionExplanation On YouTubePropositional Logic Help-Line

Q140➡ | GATE 2014 Set-2
The security system at an IT office is composed of 10 computers of which exactly four are working. To check whether the system is functional,
the officials inspect four of the computers picked at random (without replacement). The system is deemed functional if at least three of the
four computers inspected are working. Let the probability that the system is deemed functional be denoted by p. Then 100p =__________.

Show Answer With Best Explanation

Answer: 11.85 to 11.95
Explanation: Upload Soon

More DiscussionExplanation On YouTube Probability Help-Line

Q141➡ | GATE 2014 Set-2
Each of the nine words in the sentence “The quick brown fox jumps over the lazy dog” is written on a separate piece of paper. These nine pieces of paper are kept in a box. One of the pieces is drawn at random from the box. The expected length of the word drawn is _____________. (The answer should be rounded to one decimal place.)

Show Answer With Best Explanation

Answer: 3.8 to 3.9
Explanation: Upload Soon

More DiscussionExplanation On YouTubeProbabbility Help-Line

Q142➡ | GATE 2014 Set-2
The maximum number of edges in a bipartite graph on 12 vertices is_____.

Show Answer With Best Explanation

Answer: 36
Explanation: Upload Soon

More DiscussionExplanation On YouTubeGraph-TheoryHelp-Line

Q143➡ | GATE 2014 Set-2
If the matrix A is such that
then the determinant of A is equal to .

Show Answer With Best Explanation

Answer: 0
Explanation: Upload Soon

More DiscussionExplanation On YouTube Linear-Algebra Help-Line

Q144➡ | GATE 2014 Set-2
A non-zero polynomial ƒ(x) of degree 3 has roots at x = 1, x = 2 and x = 3. Which one of the following must be TRUE?
i ➥ ƒ(0)ƒ(4) < 0
ii ➥ ƒ(0)ƒ(4) > 0
iii ➥ ƒ(0) + ƒ(4) > 0
iv ➥ ƒ(0) + ƒ(4) < 0

Show Answer With Best Explanation

Answer: I
Explanation: Upload Soon

More DiscussionExplanation On YouTubeCalculus Help-Line

Q145➡ | GATE 2014 Set-2
Consider the tree arcs of a BFS traversal from a source node W in an unweighted, connected, undirected graph. The tree T formed by the tree arcs is a data structure for computing
i ➥ the shortest path between every pair of vertices.
ii ➥ the shortest path from W to every vertex in the graph.
iii ➥ the shortest paths from W to only those nodes that are leaves of T
iv ➥ the longest path in the graph.

Show Answer With Best Explanation

Answer: II
Explanation: Upload Soon

More DiscussionExplanation On YouTube graphHelp-Line

Q146➡ | GATE 2014 Set-2
In the Newton-Raphson method, an initial guess of x0 = 2 is made and the sequence x0, x1, x2 … is obtained for the function
0.75x3 – 2x2 – 2x + 4 = 0

Consider the statements
(I) x3 = 0.
(II) The method converges to a solution in a finite number of iterations.
Which of the following is TRUE?
i ➥ Only I
ii ➥ Only II
iii ➥ Both I and II
iv ➥ Neither I nor II

Show Answer With Best Explanation

Answer: I
Explanation: Upload Soon

More DiscussionExplanation On YouTube Newton-RaphsonHelp-Line

Q147➡ | GATE 2014 Set-2
The product of the non-zero eigenvalues of the matrix

is_______.

Show Answer With Best Explanation

Answer: 6
Explanation: Upload Soon

More DiscussionExplanation On YouTubeMatrixHelp-Line

Q148➡ | GATE 2014 Set-2
The probability that a given positive integer lying between 1 and 100 (both inclusive) is NOT divisible by 2, 3 or 5 is____.

Show Answer With Best Explanation

Answer: 0.259 To 0.261
Explanation: Upload Soon

More DiscussionExplanation On YouTubeProbability Help-Line

Q149➡ | GATE 2014 Set-2
The number of distinct positive integral factors of 2014 is______.

Show Answer With Best Explanation

Answer: 8
Explanation: Upload Soon

More DiscussionExplanation On YouTube Set Theory Help-Line

Q150➡ | GATE 2014 Set-2
Consider the following relation on subsets of the set S of integers between 1 and 2014. For two distinct subsets U and V of S we say U < V if the minimum element in the symmetric difference of the two sets is in U.
Consider the following two statements:
S1: There is a subset of S that is larger than every other subset.
S2: There is a subset of S that is smaller than every other subset. Which one of the following is CORRECT?
i ➥ Both S1 and S2 are true
ii ➥ S1 is true and S2 is false
iii ➥ S2 is true and S1 is false
iv ➥ Neither S1 nor S2 is true

Show Answer With Best Explanation

Answer: I
Explanation: Upload Soon

More DiscussionExplanation On YouTubeRelation Help-Line

Q151➡ | GATE 2014 Set-2
A cycle on n vertices is isomorphic to its complement. The value of n is________.

Show Answer With Best Explanation

Answer: 5
Explanation: Upload Soon

More DiscussionExplanation On YouTubeGraphHelp-Line

Q152➡ | GATE 2014 Set-3
Consider the following statements:
P: Good mobile phones are not cheap
Q: Cheap mobile phones are not good

L: P implies Q
M: Q implies P
N: P is equivalent to Q
Which one of the following about L, M, and N is CORRECT?
i ➥ Only L is TRUE.
ii ➥ Only M is TRUE.
iii ➥ Only N is TRUE.
iv ➥ L, M and N are TRUE.

Show Answer With Best Explanation

Answer: IV
Explanation: Upload Soon

More DiscussionExplanation On YouTube Prepositional Logic Help-Line

Q153➡ | GATE 2014 Set-3
Let X and Y be finite sets and f: X→Y be a function. Which one of the following statements is TRUE?
i ➥ For any subsets A and B of X, |f(A ∪ B)| = |f(A)|+|f(B)|
ii ➥ For any subsets A and B of X, f(A ∩ B) = f(A) ∩ f(B)
iii ➥ For any subsets A and B of X, |f(A ∩ B)| = min{ |f(A)|,f|(B)|}
iv ➥ For any subsets S and T of Y, f -1 (S ∩ T) = f -1 (S) ∩ f -1 (T)

Show Answer With Best Explanation

Answer: IV
Explanation: Upload Soon

More DiscussionExplanation On YouTube Set Theory Help-Line

Q154➡ | GATE 2014 Set-3
Let G be a group with 15 elements. Let L be a subgroup of G. It is known that L ≠ G and that the size of L is at least 4. The size of L is ________.

Show Answer With Best Explanation

Answer: 5
Explanation: Upload Soon

More DiscussionExplanation On YouTube Set Theory Help-Line

Q155➡ | GATE 2014 Set-3
Which one of the following statements is TRUE about every n × n matrix with only real eigenvalues?
i ➥ If the trace of the matrix is positive and the determinant of the matrix is negative, at least one of its eigenvalues is negative.
ii ➥ If the trace of the matrix is positive, all its eigenvalues are positive.
iii ➥ If the determinant of the matrix is positive, all its eigenvalues are positive.
iv ➥ If the product of the trace and determinant of the matrix is positive, all its eigenvalues are positive.

Show Answer With Best Explanation

Answer: I
Explanation: Upload Soon

More DiscussionExplanation On YouTube Linear Algebra Help-Line

Q156➡ | GATE 2014 Set-3
If V1 and V2 are 4-dimensional subspace of a 6-dimensional vector space V, then the smallest possible dimension of V1∩V2 is ______.

Show Answer With Best Explanation

Answer: 2
Explanation: Upload Soon

More DiscussionExplanation On YouTube Set Theory Help-Line

Q157➡ | GATE 2014 Set-3

Show Answer With Best Explanation

Answer: 4
Explanation: Upload Soon

More DiscussionExplanation On YouTubeCalculus Help-Line

Q158➡ | GATE 2014 Set-3
With respect to the numerical evaluation of the definite integral, K = where a and b are given,
which of the following statements is/are TRUE?
I) The value of K obtained using the trapezoidal rule is always greater than or equal to the exact value of the definite integral.
II) The value of K obtained using the Simpson’s rule is always equal to the exact value of the definite integral.
i ➥ I only
ii ➥ II only
iii ➥ Both I and II
iv ➥ Neither I nor II

Show Answer With Best Explanation

Answer: III
Explanation: Upload Soon

More DiscussionExplanation On YouTubeCalculus Help-Line

Q159➡ | GATE 2014 Set-3
The value of the integral given below is
i ➥ −2n
ii ➥ n
iii ➥ -n
iv ➥ 2n

Show Answer With Best Explanation

Answer: I
Explanation: Upload Soon

More DiscussionExplanation On YouTubeLinear Algebra Help-Line

Q159➡ | GATE 2014 Set-3
Let S be a sample space and two mutually exclusive events A and B be such that A ∪ B = S. If P(∙) denotes the probability of the event, the maximum value of P(A)P(B) is .

Show Answer With Best Explanation

Answer: 0.25
Explanation: Upload Soon

More DiscussionExplanation On YouTube ProbabilityHelp-Line

Q160➡ | GATE 2014 Set-3
Consider the set of all functions ƒ: {0,1, … ,2014} → {0,1, … ,2014} such that ƒ(ƒ(i)) = i,
for all 0 ≤ i ≤ 2014. Consider the following statements:
P. For each such function it must be the case that for every i, ƒ(i) = i.
Q. For each such function it must be the case that for some i, ƒ(i) = i.
R. Each such function must be onto.
Which one of the following is CORRECT?
i ➥ P, Q and R are true
ii ➥ Only Q and R are true
iii ➥ Only P and Q are true
iv ➥ Only R is true

Show Answer With Best Explanation

Answer: II
Explanation: Upload Soon

More DiscussionExplanation On YouTubeRelationsHelp-Line

Q161➡ | GATE 2014 Set-3
There are two elements x, y in a group (G,∗) such that every element in the group can be written as a product of some number of x’s and y’s in some order. It is known that
x ∗ x = y ∗ y = x ∗ y ∗ x ∗ y = y ∗ x ∗ y ∗ x = e
where e is the identity element. The maximum number of elements in such a group is_________.
.

Show Answer With Best Explanation

Answer: 4
Explanation: Upload Soon

More DiscussionExplanation On YouTube groups Help-Line

Q162➡ | GATE 2014 Set-3
If G is a forest with n vertices and k connected components, how many edges does G have?
i ➥
ii ➥
iii ➥
iv ➥

Show Answer With Best Explanation

Answer: III
Explanation: Upload Soon

More DiscussionExplanation On YouTubeGraph Theory Help-Line

Q163➡ | GATE 2014 Set-3
Let δ denote the minimum degree of a vertex in a graph. For all planar graphs on n vertices with δ ≥ 3, which one of the following is TRUE?
i ➥ In any planar embedding, the number of faces is at least n/2+2
ii ➥ In any planar embedding, the number of faces is less than n/2+2
iii ➥ There is a planar embedding in which the number of faces is less than n/2+2
iv ➥ There is a planar embedding in which the number of faces is at most n/ δ +1

Show Answer With Best Explanation

Answer: I
Explanation: Upload Soon

More DiscussionExplanation On YouTube Graph Theory Help-Line

Q164➡ | GATE 2014 Set-3
The CORRECT formula for the sentence, “not all rainy days are cold” is
i ➥ ∀d (Rainy(d) ∧ ∼ Cold(d))
ii ➥ ∀d ( ∼ Rainy(d) → Cold(d))
iii ➥ ∃d ( ∼ Rainy(d) → Cold(d))
iv ➥ ∃d (Rainy(d) ∧ ∼Cold(d))

Show Answer With Best Explanation

Answer: IV
Explanation: Upload Soon

More DiscussionExplanation On YouTubePropositional Logic Help-Line

Q165➡ | GATE 2013
A binary operation ⊕ on a set of integers is defined as x ⊕ y = x2 + y2. Which one of the following statements is TRUE about ⊕?
i ➥ Commutative but not associative
ii ➥ Both commutative and associative
iii ➥ Associative but not commutative
iv ➥ Neither commutative nor associative

Show Answer With Best Explanation

Answer: I
Explanation: Upload Soon

More DiscussionExplanation On YouTube Set Theory Help-Line

Q166➡ | GATE 2013
Suppose p is the number of cars per minute passing through a certain road junction between 5 PM and 6 PM, and p has a Poisson distribution with mean 3. What is the probability of observing fewer than 3 cars during any given minute in this interval?
i ➥ 8/(2e3)
ii ➥ 9/(2e3)
iii ➥ 17/(2e3)
iv ➥ 26/(2e3)

Show Answer With Best Explanation

Answer: III
Explanation: Upload Soon

More DiscussionExplanation On YouTube Probability Help-Line

Q167➡ | GATE 2013
Which one of the following does NOT equal to
i ➥
ii ➥
iii ➥
iv ➥

Show Answer With Best Explanation

Answer: I
Explanation: Upload Soon

More DiscussionExplanation On YouTube Linear Algebra Help-Line

Q168➡ | GATE 2013
Which one of the following functions is continuous at x = 3?
i ➥
ii ➥
iii ➥
iv ➥

Show Answer With Best Explanation

Answer: I
Explanation: Upload Soon

More DiscussionExplanation On YouTubeCalculus Help-Line

Q169➡ | GATE 2013
Function f is known at the following points:

The value of computed using the trapezoidal rule is
i ➥ 8.983
ii ➥ 9.003
iii ➥ 9.017
iv ➥ 9.045

Show Answer With Best Explanation

Answer: IV
Explanation: Upload Soon

More DiscussionExplanation On YouTubeTrapezoidal Rule Help-Line

Q170➡ | GATE 2013
Consider an undirected random graph of eight vertices. The probability that there is an edge between a pair of vertices is 1/2. What is the expected number of unordered cycles of length three?
i ➥ 1/8
ii ➥ 1
iii ➥ 7
iv ➥ 8

Show Answer With Best Explanation

Answer: III
Explanation: Upload Soon

More DiscussionExplanation On YouTube Graph Theory Help-Line

Q171➡ | GATE 2013
Which of the following statements is/are TRUE for undirected graphs?
P: Number of odd degree vertices is even.
Q: Sum of degrees of all vertices is even.
i ➥ P only
ii ➥ Q only
iii ➥ Both P and Q
iv ➥ Neither P nor Q

Show Answer With Best Explanation

Answer: III
Explanation: Upload Soon

More DiscussionExplanation On YouTube Graph Theory Help-Line

Q172➡ | GATE 2013
The line Graph L(G) of a simple graph G is defined as follows:
. There is exactly one vertex v(e) in L(G) for each edge e in G
. For any two edge e and e’ in G, L(G) has an edge between v(e) and v(e’), if and only if e and e’ are incident with the same vertex in G.

Which of the following statements is/ are TRUE?
(P) The line Graph of a cycle is a cycle.
(Q) The line Graph of a clique is a clique.
(R) The line Graph of a planar graph is planar.
(S) The line Graph of a Tree is Tree.
i ➥ P only
ii ➥ P and R only
iii ➥ R only
iv ➥ P, Q and S only

Show Answer With Best Explanation

Answer: I
Explanation: Upload Soon

More DiscussionExplanation On YouTube Graph Theory Help-Line

Q173➡ | GATE 2013
What is the logical translation of the following statement?
“None of my friends are perfect.”
i ➥ ∃x(F(x)∧¬P(x))
ii ➥ ∃x(¬F(x)∧P(x))
iii ➥ ∃x(¬F(x)∧¬P(x))
iv ➥ ¬∃x(F(x)∧P(x))

Show Answer With Best Explanation

Answer: IV
Explanation: Upload Soon

More DiscussionExplanation On YouTube Prepositional Logic Help-Line

Q174➡ | GATE 2013
Which one of the following is NOT logically equivalent to ¬∃x(∀y(α) ∧ ∀z(β))?
i ➥ ∀x(∃z(¬β)→∀y(α))
ii ➥ ∀x(∀z(β)→∃y(¬α))
iii ➥ ∀x(∀y(α)→∃z(¬β))
iv ➥ ∀x(∃y(¬α)→∃z(¬β))

Show Answer With Best Explanation

Answer: IV
Explanation: Upload Soon

More DiscussionExplanation On YouTubePrepositional Logic Help-Line

Q175➡ | GATE 2012
Consider the following logical inferences.
I1:
If it rains then the cricket match will not be played.
The cricket match was played.
Inference: There was no rain.

I2:
If it rains then the cricket match will not be played.
It did not rain.
Inference: The cricket match was played.

Which of the following is TRUE?
i ➥ Both I1 and I2 are correct inferences
ii ➥ I1 is correct but I2 is not a correct inference
iii ➥ I1 is not correct but I2 is a correct inference
iv ➥ Both I1 and I2 are not correct inferences

Show Answer With Best Explanation

Answer: II
Explanation: Upload Soon

More DiscussionExplanation On YouTube Propositional Logic Help-Line

Q176➡ | GATE 2012
Consider the function f(x) = sin(x) in the interval x ∈ [π/4, 7π/4]. The number and location(s) of the local minima of this function are
i ➥ One, at π/2
ii ➥ One, at 3π/2
iii ➥ Two, at π/2 and 3π/2
iv ➥ Two, at π/4 and 3π/2

Show Answer With Best Explanation

Answer: IV
Explanation: Upload Soon

More DiscussionExplanation On YouTube Calculus Help-Line

Q177➡ | GATE 2012
Let A be the 2×2 matrix with elements a11 = a12 = a21 = +1 and a22 = -1. Then the eigenvalues of the matrix A19 are
i ➥ 1024 and -1024
ii ➥ 1024√2 and -1024√2
iii ➥ 4√2 and -4√2
iv ➥ 512√2 and -512√2

Show Answer With Best Explanation

Answer: IV
Explanation: Upload Soon

More Discussion Explanation On YouTube Linear Algebra Help-Line

Q178➡ | GATE 2012
What is the correct translation of the following statement into mathematical logic?
“Some real numbers are rational”
i ➥ ∃x (real(x) ∨ rational(x))
ii ➥ ∀x (real(x) → rational(x))
iii ➥ ∃x (real(x) ∧ rational(x))
iv ➥ ∃x (rational(x) → real(x))

Show Answer With Best Explanation

Answer: III
Explanation: Upload Soon

More Discussion Explanation On YouTube Propositional Logic Help-Line

Q179➡ | GATE 2012
Let G be a simple undirected planar graph on 10 vertices with 15 edges. If G is a connected graph, then the number of bounded faces in any embedding of G on the plane is equal to
i ➥ 3
ii ➥ 4
iii ➥ 5
iv ➥ 6

Show Answer With Best Explanation

Answer: IV
Explanation: Upload Soon

More DiscussionExplanation On YouTubeGraph Theory Help-Line

Q180➡ | GATE 2012
Consider a random variable X that takes values +1 and −1 with probability 0.5 each. The values of the cumulative distribution function F(x) at x = −1 and +1 are
i ➥ 0 and 0.5
ii ➥ 0 and 1
iii ➥ 0.5 and 1
iv ➥ 0.25 and 0.75

Show Answer With Best Explanation

Answer: III
Explanation: Upload Soon

More DiscussionExplanation On YouTubeProbability Help-Line

Q181➡ | GATE 2012
Which of the following graphs is isomorphic to
i ➥
ii ➥
iii ➥
iv ➥

Show Answer With Best Explanation

Answer: II
Explanation: Upload Soon

More DiscussionExplanation On YouTube Graph Theory Help-Line

Q182➡ | GATE 2012
The bisection method is applied to compute a zero of the function f(x) = x4 – x3 – x2 – 4 in the interval [1,9]. The method converges to a solution after ______ iterations.
i ➥ 1
ii ➥ 3
iii ➥ 5
iv ➥ 7

Show Answer With Best Explanation

Answer: II
Explanation: Upload Soon

More DiscussionExplanation On YouTubeNumerical Method Help-Line

Q183➡ | GATE 2012
Suppose a fair six-sided die is rolled once. If the value on the die is 1, 2, or 3, the die is rolled a second time. What is the probability that the sum total of values that turn up is at least 6?
i ➥ 10/21
ii ➥ 5/12
iii ➥ 2/3
iv ➥ 1/6

Show Answer With Best Explanation

Answer: II
Explanation: Upload Soon

More DiscussionExplanation On YouTubeProbability Help-Line

Q184➡ | GATE 2012
How many onto (or surjective) functions are there from an n-element (n ≥ 2) set to a 2-element set?
i ➥ 2n
ii ➥ 2n-1
iii ➥ 2n-2
iv ➥ 2(2n– 2)

Show Answer With Best Explanation

Answer: III
Explanation: Upload Soon

More DiscussionExplanation On YouTube Function Help-Line

Q185➡ | GATE 2012
Let G be a complete undirected graph on 6 vertices. If vertices of G are labeled, then the number of distinct cycles of length 4 in G is equal to
i ➥ 15
ii ➥ 30
iii ➥ 45
iv ➥ 360

Show Answer With Best Explanation

Answer: Marks to All
Explanation: Upload Soon

More DiscussionExplanation On YouTubeGraph Theory Help-Line

Q186➡ | GATE 2011
If two fair coins are flipped and at least one of the outcomes is known to be a head, what is the probability that both outcomes are heads?
i ➥ 1/3
ii ➥ 1/4
iii ➥ 1/2
iv ➥ 2/3

Show Answer With Best Explanation

Answer: I
Explanation: Upload Soon

More DiscussionExplanation On YouTube Probability Help-Line


Q187➡ | GATE 2011
K4 and Q3 are graphs with the following structures.

Which one of the following statement is TRUE in relation to these graphs?
i ➥ K4 is planar while Q3 is not
ii ➥ Both K4 and Q3 are planar
iii ➥ Q3 is planar while K4 is not
iv ➥ Neither K4 not Q3 is planar

Show Answer With Best Explanation

Answer: II
Explanation: Upload Soon

More DiscussionExplanation On YouTubeGraph Theory Help-Line

Q188➡ | GATE 2011
If the difference between the expectation of the square of a random variable (E[X2]) and the square of the expectation of the random variable (E[X])2 is denoted by R, then
i ➥ R = 0
ii ➥ R < 0
iii ➥ R ≥ 0
iv ➥ R > 0

Show Answer With Best Explanation

Answer: III
Explanation: Upload Soon

More DiscussionExplanation On YouTubeProbability Help-Line

Q189➡ | GATE 2011
Which one of the following options is CORRECT given three positive integers x,y and z, and a predicate

P(x) = ¬(x=1)∧∀y(∃z(x=y*z) ⇒ (y=x)∨(y=1))
i ➥ P(x) being true means that x is a prime number
ii ➥ P(x) being true means that x is a number other than 1
iii ➥ P(x) is always true irrespective of the value of x
iv ➥ P(x) being true means that x has exactly two factors other than 1 and x

Show Answer With Best Explanation

Answer: I
Explanation: Upload Soon

More DiscussionExplanation On YouTube Propositional Logic Help-Line

Q190➡ | GATE 2011
Given i=√-1, what will be the evaluation of the definite integral
i ➥ 0
ii ➥ 2
iii ➥ -i
iv ➥ i

Show Answer With Best Explanation

Answer: IV
Explanation: Upload Soon

More DiscussionExplanation On YouTube Calculus Help-Line

Q191➡ | GATE 2011
Consider a finite sequence of random values X = [x1, x2, …, xn]. Let μx be the mean and σx be the standard deviation of X. Let another finite sequence Y of equal length be derived from this as yi = a * xi + b, where a and b are positive constants. Let μy be the mean and σy be the standard deviation of this sequence. Which one of the following statements is INCORRECT?
i ➥ Index position of mode of X in X is the same as the index position of mode of Y in Y.
ii ➥ Index position of median of X in X is the same as the index position of median of Y in Y.
iii ➥ μy = aμx + b
iv ➥ σy = aσx + b

Show Answer With Best Explanation

Answer: IV
Explanation: Upload Soon

More DiscussionExplanation On YouTubeProbability Help-Line

Q192➡ | GATE 2011 GATE 2011
A deck of 5 cards (each carrying a distinct number from 1 to 5) is shuffled thoroughly. Two cards are then removed one at a time from the deck. What is the probability that the two cards are selected with the number on the first card being one higher than the number on the second.
i ➥ 1/5
ii ➥ 4/25
iii ➥ 1/4
iv ➥ 2/5

Show Answer With Best Explanation

Answer: I
Explanation: Upload Soon

More DiscussionExplanation On YouTube Probability Help-Line

Q193➡ | GATE 2011
Consider the matrix as given below.

Which one of the following provides the CORRECT values of eigenvalues of the matrix?
i ➥ 1, 4, 3
ii ➥ 3, 7, 3
iii ➥ 7, 3, 2
iv ➥ 1, 2, 3

Show Answer With Best Explanation

Answer: I
Explanation: Upload Soon

More DiscussionExplanation On YouTubeLinear Algebra Help-Line

Q194➡ | GATE 2010
Let G = (V,E) be a graph. Define ξ(G) = , where id is the number of vertices of degree d in G. If S and T are two different trees with ξ(S) = ξ(T),then
i ➥ |S| = 2|T|
ii ➥ |S| = |T| – 1
iii ➥ |S| = |T|
iv ➥ |S| = |T| + 1

Show Answer With Best Explanation

Answer: III
Explanation: Upload Soon

More DiscussionExplanation On YouTube Graph Theory Help-Line

Q195➡ | GATE 2010
Newton-Raphson method is used to compute a root of the equation x2 – 13 = 0 with 3.5 as the initial value. The approximation after one iteration is
i ➥ 3.575
ii ➥ 3.676
iii ➥ 3.667
iv ➥ 3.607

Show Answer With Best Explanation

Answer: IV
Explanation: Upload Soon

More DiscussionExplanation On YouTube Newton-Raphson methodHelp-Line

Q196➡ | GATE 2010
What is the possible number of reflexive relations on a set of 5 elements?
i ➥ 210
ii ➥ 215
iii ➥ 220
iv ➥ 225

Show Answer With Best Explanation

Answer: III
Explanation: Upload Soon

More DiscussionExplanation On YouTube Set Theory Help-Line

Q197➡ | GATE 2010
Consider the set S = {1, ω, ω2}, where ω and ω2 are cube roots of unity. If * denotes the multiplication operation, the structure (S,*) forms
i ➥ A group
ii ➥ A ring
iii ➥ An integral domain
iv ➥ A field

Show Answer With Best Explanation

Answer: I
Explanation: Upload Soon

More DiscussionExplanation On YouTube Set Theory Help-Line

Q198➡ | GATE 2010
What is the value of
i ➥ 0
ii ➥ e-2
iii ➥ e-1/2
iv ➥ 1

Show Answer With Best Explanation

Answer: II
Explanation: Upload Soon

More DiscussionExplanation On YouTube Calculus Help-Line

Q199➡ | GATE 2010
Consider a company that assembles computers. The probability of a faulty assembly of any computer is p. The company therefore subjects each computer to a testing process. This testing process gives the correct result for any computer with a probability of q. What is the probability of a computer being declared faulty?
i ➥ pq + (1 – p)(1 – q)
ii ➥ (1 – q)p
iii ➥ (1 – p)q
iv ➥ pq

Show Answer With Best Explanation

Answer: I
Explanation: Upload Soon

More DiscussionExplanation On YouTube ProbabilityHelp-Line

Q200➡ | GATE 2010
What is the probability that divisor of 1099 is a multiple of 1096?
i ➥ 1/625
ii ➥ 4/625
iii ➥ 12/625
iv ➥ 16/625

Show Answer With Best Explanation

Answer: I
Explanation: Upload Soon

More DiscussionExplanation On YouTube ProbabilityHelp-Line

Q201➡ | GATE 2010
The degree sequence of a simple graph is the sequence of the degrees of the nodes in the graph in decreasing order. Which of the following sequences can not be the degree sequence of any graph?
(I) 7, 6, 5, 4, 4, 3, 2, 1
(II) 6, 6, 6, 6, 3, 3, 2, 2
(III) 7, 6, 6, 4, 4, 3, 2, 2
(IV) 8, 7, 7, 6, 4, 2, 1, 1
i ➥ I and II
ii ➥ III and IV
iii ➥ IV only
iv ➥ II and IV

Show Answer With Best Explanation

Answer: IV
Explanation: Upload Soon

More DiscussionExplanation On YouTubeGraph TheoryHelp-Line

Q202➡ | GATE 2010
Consider the following matrix If the eigenvalues of A are 4 and 8, then
i ➥ x=4, y=10
ii ➥ x=5, y=8
iii ➥ x=-3, y=9
iv ➥ x=-4, y=10

Show Answer With Best Explanation

Answer: IV
Explanation: Upload Soon

More DiscussionExplanation On YouTubeLinear AlgebraHelp-Line

Q203➡ | GATE 2010
Suppose the predicate F(x, y, t) is used to represent the statement that person x can fool person y at time t. Which one of the statements below expresses best the meaning of the formula ∀x∃y∃t(¬F(x, y, t))?
i ➥ Everyone can fool some person at some time
ii ➥ No one can fool everyone all the time
iii ➥ Everyone cannot fool some person all the time
iv ➥ No one can fool some person at some time

Show Answer With Best Explanation

Answer: II
Explanation: Upload Soon

More DiscussionExplanation On YouTube Propositional Logic Help-Line

error: Content is protected !!
Open chat
1
Hi,how Can We Help You ?